What is the Pattern for Cumulative Sums in a Summations Type Problem?

  • Thread starter Thread starter Jamezy
  • Start date Start date
  • Tags Tags
    Type
Click For Summary
The discussion centers on a mathematical problem involving cumulative sums after sequential deletions of numbers. Initially, the sequence of natural numbers is modified by deleting every third number, resulting in a new sequence whose cumulative sums yield cubic numbers. Participants suggest analyzing the differences between successive cumulative sums to identify a pattern, specifically focusing on the differences 6, 12, 18, and 24. This approach aims to establish a proof for why the cumulative sums ultimately correspond to cubic numbers. The conversation emphasizes working both forwards and backwards to solidify understanding of the underlying mathematical principles.
Jamezy
Messages
1
Reaction score
0

Homework Statement



Write down the numbers 1,2,3, ….
Delete every third number, beginning
with the third. Write down the
cumulative sums of the numbers which
remain. That is:
1 2 3 4 5 6 7 …
1 2 4 5 7 …
1 3 7 12 19 …
Now delete every second number,
starting with the second, and write
down the cumulative sums of what
remains


I know that it always ends up as the cubic numbers ie:

1 8 27 64 etc

But how would I make a proof of this?


Homework Equations



Summations of r, r^2 and 1 between 1 and n



Literally don'tknow how to do it at all!
 
Physics news on Phys.org
Try working forwards and backwards from the sequence just before you did the last cumulative sum yielding the cubes. That was 1,7,19,37,61,... Look at difference between successive elements. That gives you 6,12,18,24,... there's a pretty obvious pattern there. Can you work forward to show the successive sums of that are the cubes? Now can you look back and see how to prove how those differences come about?
 
Question: A clock's minute hand has length 4 and its hour hand has length 3. What is the distance between the tips at the moment when it is increasing most rapidly?(Putnam Exam Question) Answer: Making assumption that both the hands moves at constant angular velocities, the answer is ## \sqrt{7} .## But don't you think this assumption is somewhat doubtful and wrong?

Similar threads

  • · Replies 7 ·
Replies
7
Views
1K
  • · Replies 23 ·
Replies
23
Views
4K
  • · Replies 16 ·
Replies
16
Views
2K
  • · Replies 12 ·
Replies
12
Views
3K
  • · Replies 1 ·
Replies
1
Views
2K
  • · Replies 1 ·
Replies
1
Views
2K
Replies
2
Views
1K
  • · Replies 2 ·
Replies
2
Views
1K
  • · Replies 1 ·
Replies
1
Views
2K
  • · Replies 13 ·
Replies
13
Views
2K